You are on page 1of 8

Chapter 4 (Laplace transforms): Solutions

(The table of Laplace transforms is used throughout.)

Solution 4.1(a)
1
y
j
Hsin H4 tL cos H2 tLL = i
z
j sin H4 tLz
{
k2
1
= H sin H4 tLL
2
1
4
=
2
2 s + 16
2
= .
2
s + 16

Solution 4.1(b)
2
y
ii1
z
t
-t y
z
j
z
j
H cosh 2 HtL L = j

L
z
j
z
j
z
j 2
{ {
kk

1
-2 t y
i 2 t
j
z
=j
j + + z
z
2
4 {
k 4

1
1
1
1
1
1
= + +
4
s - 2
2
s
4
s + 2
s2 - 2
=
.
s Hs2 - 4L

Solution 4.1(c)
1
y
j
Hcos Ha tL sinh Ha tLL = i
z
j Ha t - -a t L cos Ha tLz
{
k2
1
= Ha t cos Ha tL - -a t cos Ha tL L
2
1 i
s-a
s+a
j
= j
-

2
2
2 j
Hs
aL
+
a
Hs
+
aL2 + a2
k
a s2 - 2 a3
=
.
s4 + 4 a4

Solution 4.1(d)
2
Ht2 -3 t L = H-3 t t2 L =
.
Hs + 3L3

y
z
z
z
{

Solution 4.2(a)
Use partial fractions, i.e. set
s
A
B

= +
Hs + 3L Hs + 5L
Hs + 3L
Hs + 5L

s = A Hs + 5L + B Hs + 3L

Using the coverup method for example we see that


3
5
A = - , B = .
2
2

Hence
s
3
1
5
1

= - +
Hs + 3L Hs + 5L
2
Hs + 3L
2
Hs + 5L

5
3
y+i
y.
j
j
=i
z
j -5 t z
j - -3 t z
z
{
k 2
k 2
{

And so the inverse Laplace transform


s
i
-1 j

j
k Hs + 3L Hs + 5L

3
5
y
z
z = - -3 t + -5 t .
2
2
{

Solution 4.2(b)
Use partial fractions, set
1
A
Bs+C

= +
2
2
s Hs + k L
s
s2 + k2

1 = A Hs2 + k2 L + HB s + CL s

1 = A s2 + A k2 + B s2 + C s

1 = HA + BL s2 + C s + A k2

Equating coefficients of
s2 :

A+B=0

s1 :

C=0

s0 :

1
A =
k2

1
Using the first equation we see that B = -
k2 , and hence

1
1
1
1
s

= -

2
2
2
2
2
s Hs + k L
k
s
k
Hs + k2 L
s
1 i1
y
z
= j
j -
z
2
2
Hs + k2 L {
k ks

1
= H1 - cos Hk tLL.
k2

And so the inverse Laplace transform


1
1
i
y
z
H1 - cos Hk tLL.
-1 j
j
z =
2
2
k2
k s Hs + k L {

Solution 4.2(c)
1
, note that from the table of
To find the inverse Laplace transform of
Hs + 3L2
1
Laplace transforms, the Laplace transform of t is
s 2 , and so if we apply the
1
-3
t
. Hence
shift theorem, the Laplace transform of
t must be
Hs + 3L2

1
i
y
-3 t
j
z
z
.
-1 j
j
z=t
2
k Hs + 3L {

Solution 4.3
To solve the initial value problem
y + y = t, y H0L = 0, y H0L = 2;

we take the Laplace transform of both sides of the differential equation


Hy HtL + y HtLL = HtL

Hy HtLL + Hy HtLL = HtL

1
s2
y HsL - s y H0L - y H0L +
y HsL =
s2
1
s2
y HsL - 2 +
y HsL =
s2
1
s2
y HsL +
y HsL = + 2
s2
1
Hs2 + 1L
y HsL = + 2
s2
1
2

y HsL =
+
.
s2 H s2 + 1L
H s2 + 1L

Using partial fractions, we set


1
A
B

= +

2
2
2
2
s H s + 1L
s
H s + 1L

Using the coverup method (first set s=0 and then s2 =1) we find that A = 1,
B = -1. Hence
1
1
2

y HsL = -
+

s2
Hs2 + 1L
Hs2 + 1L

1
1

y HsL = +

2
2
s
Hs + 1L
y HtL = t + sin HtL.

Solution 4.4
To solve the initial value problem
y + 2 y + y = 3 t e- t , y H0L = 4, y H0L = 2;

we take the Laplace transform of both sides of the differential equation


Hy HtL + 2 y HtL + y HtLL = H3 t e- t L

Hy HtLL + 2 Hy HtLL + Hy HtLL = 3 Ht e- t L


1
s2
y HsL - s y H0L - y H0L + 2 Hs
y HsL - y H0LL +
y HsL = 3

Hs + 1L2
1
s2
y HsL - 4 s - 2 + 2 Hs
y HsL - 4L +
y HsL = 3

Hs + 1L2
1
s2
y HsL + 2 s
y HsL +
y HsL = 3
+ 10 + 4 s
Hs + 1L2
1
Hs2 + 2 s + 1L
y HsL = 3
+ 10 + 4 s
Hs + 1L2
1
Hs + 1L2
y HsL = 3
+ 10 + 4 s
Hs + 1L2
1
10 + 4 s

y HsL = 3
+
.
4
Hs + 1L
Hs + 1L2

Now notice that


10 + 4 s
4s+4+6
4 Hs + 1L + 6
4
6

=
=
= +

2
2
2
Hs
+
1L
Hs + 1L
Hs + 1L
Hs + 1L
Hs + 1L2

Hence

1
1
1

y HsL = 3
+ 4 + 6

4
Hs
+
1L
Hs + 1L
Hs + 1L2

1
y HtL = -t t3 + 4 - t + 6 - t t.
2

Solution 4.5
To solve the initial value problem
y + 16 y = 32 t,

y H0L = 3, y H0L = -2;

we take the Laplace transform of both sides of the differential equation

Hy HtL + y HtLL = H32 tL

Hy HtLL + Hy HtLL = 32 HtL

1
s2
y HsL - s y H0L - y H0L + 16
y HsL = 32
s2
1
s2
y HsL - 3 s + 2 + 16
y HsL = 32
s2
32
Hs2 + 16L
y HsL = - 2 + 3 s
s2
32
2
3s

y HsL =
-
+

s2 Hs2 + 16L
Hs2 + 16L
Hs2 + 16 L
2
2
2
3s

y HsL = -
-
+

2
2
2
2
s
Hs + 16 L
Hs + 16L
Hs + 16 L
2
4
3s

y HsL = -
+

2
2
2
s
Hs + 16L
Hs + 16 L
y HtL = 2 t - sin H4 tL + 3 cos H4 tL.

32
.
Note that we used partial fractions to split the term
s2 Hs2 +16L

Solution 4.6
To solve the initial value problem
y - 3 y + 2 y = 4,

y H0L = 1, y H0L = 0;

we take the Laplace transform of both sides of the differential equation

Hy - 3 y + 2 yL = H4L

Hy HtLL - 3 Hy HtLL + 2 Hy HtLL = H4L

4
s2
y HsL - s y H0L - y H0L - 3 Hs
y HsL - y H0LL + 2
y HsL =
s
4
s2
y HsL - s - 0 - 3 Hs
y HsL - 1L + 2
y HsL =
s
4
Hs2 - 3 s + 2L
y HsL = + s - 3
s
4
Hs - 1L Hs - 2L
y HsL = + s - 3
s

4
s-3

y HsL =

+
.
s Hs - 1L Hs - 2L
Hs - 1L Hs - 2L

Using partial fractions, set


4
A
B
C

= + +
s Hs - 1L Hs - 2L
s
Hs - 1L
Hs - 2L

4 = A Hs - 1L Hs - 2L + B s Hs - 2L + C s Hs - 1L.

Using the coverup method we soon see that


A = 2, B = -4, C = 2.

Also using partial fractions, set


s-3
D
E

= +
Hs - 1L Hs - 2L
Hs - 1L
Hs - 2L

s - 3 = D Hs - 2L + E Hs - 1L.

Using the coverup method we soon see that


D = 2, E = -1.

Hence
2
4
2
2
1

y HsL = - + + -
s
Hs - 1L
Hs - 2L
Hs - 1L
Hs - 2L

2
2
1

y HsL = - +
s
Hs - 1L
Hs - 2L

y HtL = 2 - 2 t + 2 t .

Solution 4.7
To solve the initial value problem
y + 4 y + 4 y = 6 -2 t ,

y H0L = -2, y H0L = 8;

we take the Laplace transform of both sides of the differential equation

Hy + 4 y + 4 yL = H6 -2 t L

Hy HtLL + 4 Hy HtLL + 4 Hy HtLL = 6 H-2 t L

6
s2
y HsL - s y H0L - y H0L + 4 Hs
y HsL - y H0LL + 4
y HsL =
s+2
6
s2
y HsL + 2 s - 8 + 4 Hs
y HsL + 2L + 4
y HsL =
s+2
6
Hs2 + 4 s + 4L
y HsL = - 2 s
s+2
6
Hs + 2L2
y HsL = - 2 s
s+2

6
2s

y HsL =
-

3
Hs + 2L
Hs + 2L2

6
2 Hs + 2L - 4

y HsL =
-

3
Hs + 2L
Hs + 2L2

4
6
i 2
y

j
z
-
z
y HsL =
- j
z
j
2
s
+
2
Hs
+
2L
Hs + 2L3
k
{

6
2
4

y HsL =
- +

3
s+2
Hs + 2L
Hs + 2L2

y HtL = -2 t 3 t2 - -2 t 2 + -2 t 4 t

y HtL = -2 t H3 t2 + 4 t - 2L.

Solution 4.8
To solve the initial value problem
y + 4 y + 5 y = Ht - 4 L,

y H0L = 0, y H0L = 3;

we take the Laplace transform of both sides of the differential equation


Hy HtL + 4 y HtL + 5 y HtLL = H Ht - 4 LL

Hy HtLL + 4 Hy HtLL + 5 Hy HtLL = H Ht - 4 LL


s2
y HsL - s y H0L - y H0L + 4 Hs
y HsL - y H0LL + 5
y HsL = -4 s
s2
y HsL - 0 - 3 + 4 Hs
y HsL - 0L + 5
y HsL = -4 s
Hs2 + 4 s + 5L
y HsL = -4 s + 3

-4 s
3

y HsL =
+

2
2
s +4s+5
s +4s+5
-4 s
3

y HsL =
+

2
Hs + 2L + 1
Hs + 2L2 + 1

1
1
i
y
j
z

y HsL = -4 s j
z

j
z + 3
2
Hs + 2L2 + 1
k Hs + 2L + 1 {

y HsL = -4 s
g HsL + 3
g HsL

where
1

g HsL =
= H-2 t sin HtLL
Hs + 2L2 + 1

and
-4 s
g HsL = H-2 Ht-4 L sin Ht - 4 LL.

Hence
y HtL = 9

-2 Ht-4 L sin Ht - 4 L + 3 -2 t sin HtL,


3 -2 t sin HtL,

t > 4 ,
t 4 .

Plot@UnitStep@t - 4 D H-2 Ht-4 L Sin@t - 4 DL + 3 -2 t Sin@tD, 8t, 0, 8 <, PlotRange AllD

0.5
0.4
0.3
0.2
0.1
5
Graphics

10

15

20

25

You might also like